Đến nội dung

Secrets In Inequalities VP nội dung

Có 298 mục bởi Secrets In Inequalities VP (Tìm giới hạn từ 29-03-2020)



Sắp theo                Sắp xếp  

#382021 $k!+48=48(k+1)^{m}$

Đã gửi bởi Secrets In Inequalities VP on 30-12-2012 - 17:17 trong Số học

Có thiếu đk $k+1$ nguyên tố không nhỉ?

$k+1$ nguyên tố chỉ là $1$ TH của bài này thôi cậu .
Nếu $k+1$ là hợp số $\Rightarrow k+1= x.y$ ( $x,y\leq k$ )
$\Rightarrow k!= 1.2...x...y...k\vdots xy\Rightarrow k!\vdots k+1\Rightarrow 48\vdots k+1$.Dễ rồi !



#381865 chứng minh rằng hoặc p2 | a + b, hoặc p3 | a3 + b3

Đã gửi bởi Secrets In Inequalities VP on 30-12-2012 - 08:16 trong Số học

Cho p > 3 là số nguyên tố. Giả sử a, b là các số nguyên sao cho p | a + b và p2 | a3 + b3. Hãy chứng minh rằng hoặc p2 | a + b, hoặc p3 | a3 + b3

Vì $p^{2} | a ^{3}+ b^{3}= (a+b)(a^2-ab+b^2)$ nên chỉ xảy ra
+ $p^{2} | (a+b)$ ta có đpcm
+ $p | a^2-ab+b^2$ .Mà $p | a+b\Rightarrow p | (a+b)^2= a^2+2ab+b^2$
$\Rightarrow p | 3ab$ .Mà $p>3$ $\Rightarrow (p,3)=1\Rightarrow p | ab$
Lại có $p | a+b,p \in \mathbb{P}$ nên $p | a$ và $p | b$
$\Rightarrow p^{3} | a^{3}, p^{3} | b^{3}\Rightarrow p^{3} | a^{3}+b^3$



#381768 $(n+1)^n-1 \vdots n^2$

Đã gửi bởi Secrets In Inequalities VP on 29-12-2012 - 22:12 trong Các dạng toán THPT khác

2.Cho $a\geq 3; n> 1$ và thoả mãn $a^{n}\equiv 1 (mod n)$ với n là số tự nhiên. Gọi p là ước nguyên tố nhỏ nhất của n.
CMR : $a-1\vdots p$

Ta có :$a^{n}\equiv 1(Mod n)\Rightarrow a^{n}\equiv 1(Mod p)$
Gọi $h$ là cấp của $a$ theo mod $p$ $\Rightarrow a^{h}\equiv 1(Mod p)$
Theo tính chất của cấp thì $h|n$
Theo Fermart thì $h|p-1$
Ta thấy $h=1$ và nếu $h\neq 1$ thì gọi $q$ là ước nguyên tố nhỏ nhất của $h$ $\Rightarrow q|h\Rightarrow q|n$
Mà $q\leq h\leq p-1< p$ nên mâu thuẫn với việc $p$ là ước nguyên tố nhỏ nhất của $n$
Suy ra $h=1$ $\Rightarrow a\equiv 1(Modp)\Rightarrow a-1\vdots p$



#381696 $$3\left( {{x}^{2}}+{{...

Đã gửi bởi Secrets In Inequalities VP on 29-12-2012 - 20:16 trong Bất đẳng thức và cực trị

Cho $x,y,z$ là các số dương. Chứng minh rằng với $x+y+z=3$ thì
$$3\left( {{x}^{2}}+{{y}^{2}}+{{z}^{2}} \right)+4xyz\ge 13$$

Làm thế này cho đỡ đau đầu !
Ta thấy sẽ có ít nhất 2 trong 3 số $x,y,z$ cùng lớn hơn hoặc nhỏ hơn $1$.Giả sử là $y$ và $z$
$\Rightarrow (y-1)(z-1)\geq 0\Leftrightarrow yz\geq y+z-1= 2-x$
$\Rightarrow xyz\geq 2x-x^2\Rightarrow 4xyz\geq 8x-4x^2$
Áp dụng nhận xét trên kết hợp vs bđt cơ bản $a^2+b^2\geq \frac{(a+b)^2}{2}$ ta được :
$VT\geq 3x^2+\frac{3}{2}.(y+z)^2+8x-4x^2= \frac{3}{2}.(3-x)^2-x^2+8x$
$$= \frac{x^2}{2}-x+\frac{1}{2}+13= \frac{(x-1)^2}{2}+13\geq 13$$
Vậy ta có $Q.E.D$



#381566 Chứng minh tồn tại $k_1,k_2,...,k_s$ để $b_1+m_1k_1=b_2+m_2k_2...

Đã gửi bởi Secrets In Inequalities VP on 29-12-2012 - 14:37 trong Số học

Cho $m_1,m_2,...,m_s$ là các số tự nhiên thỏa $(m_i,m_j)=1$ với mọi $i \ne j$. Cho $b_1,b_2,...,b_s \in \mathbb{Z}$. Chứng minh tồn tại $k_1,k_2,...,k_s \in \mathbb{Z}$ sao cho:
$$b_1+m_1k_1=b_2+m_2k_2=...=b_s+m_sk_s$$

Xét hệ phương trình đồng dư : $x\equiv {b_i} (Mod{m_i})$
Theo định lí thặng dư của Tàu thì hệ luôn có nghiệm $c$.$\Rightarrow c= {b_i}+{m_i}{k_i}$ suy ra $Q.E.D$



#381559 Chứng minh $m\vdots 1979$

Đã gửi bởi Secrets In Inequalities VP on 29-12-2012 - 14:11 trong Số học

Cho m,n là các số nguyên dương sao cho:
$\frac{m}{n}=1-\frac{1}{2}+\frac{1}{3}-\frac{1}{4}+...-\frac{1}{1318}+\frac{1}{1319}$
Chứng minh $m\vdots 1979$

Ta thấy $1979$ là số nguyên tố .
Ta có : $\frac{m}{n}=1-\frac{1}{2}+\frac{1}{3}-\frac{1}{4}+...-\frac{1}{1318}+\frac{1}{1319}$
$$=(1+\frac{1}{2}+\frac{1}{3}+\frac{1}{4}+...+\frac{1}{1318}+\frac{1}{1319})-2.(\frac{1}{2}+\frac{1}{4}+...+\frac{1}{1318})$$
$$=(1+\frac{1}{2}+\frac{1}{3}+\frac{1}{4}+...+\frac{1}{1318}+\frac{1}{1319})-(1+\frac{1}{2}+...+\frac{1}{659})$$
$$=\frac{1}{660}+\frac{1}{661}...+\frac{1}{1318}+\frac{1}{1319}$$
$$=(\frac{1}{660}+\frac{1}{1319})+(\frac{1}{661}+\frac{1}{1338})+...$$
$$=\frac{1979}{660.1319}+\frac{1979}{661.1318}+...$$
$$=1979.(\frac{1}{660.1319}+\frac{1}{661.1318}+...)= 1979.\frac{p}{q}$$
trong đó $q=660.661...1319\Rightarrow (q,1979)=1$
Ta có : $\frac{m}{n}= 1979.\frac{p}{q}\Rightarrow mq=1979np\Rightarrow mq\vdots 1979$
Mà $(q,1979)=1\Rightarrow m\vdots 1979\Rightarrow Q.E.D$



#381558 Với mỗi số nguyên dương n, ký hiệu S(n) là tổng tất cả các chữ số trong biểu...

Đã gửi bởi Secrets In Inequalities VP on 29-12-2012 - 13:55 trong Số học

Với mỗi số nguyên dương n, ký hiệu S(n) là tổng tất cả các chữ số trong biểu diễn thập phân của n.
Xét các số nguyên dương m là bội của 2003. Hãy tìm giá trị nhỏ nhất của S(m).

Đặt $p=2003\in\mathbb{P}$.
+ Nếu $S(n)= 1\Rightarrow n= 100...000= 10^{^{k}}\not\vdots 2003\Rightarrow S(n)> 1$
+ Nếu $S(n)= 2$$\Rightarrow n= 200...000$ hoặc $n=100...100...00= 10^{i}+10^{j}$
Ta có $n=200...000= 2.10^{k} \not\vdots 2003$ $\Rightarrow n=10^i+10^j$
$\Rightarrow 10^i+10^j\vdots 2003=p\Rightarrow 10^i= -10^j(Modp)$
$\Rightarrow 10^{i-j}= -1(Modp)$$\Rightarrow 10^k= -1(Modp)$ ( $k=i-j$ )
Dễ thấy $2^{10}= 1024\equiv 10^7(Modp)$ , suy ra :
$(2^{5k})^{2}= 2^{10k}\equiv (10^{k})^{7}\equiv -1(Modp)$
$\Rightarrow -1$ là số chính phương mod p suy ra $p$ có dạng $4k+1$
$\Rightarrow 2003=p= 4k+1\Rightarrow k= \frac{2002}{4} \not \in \mathbb{Z}$
Suy ra vô lí $\Rightarrow S(n)> 2$
+ Bây giờ ta chứng minh là có $n$ để $S(n)= 3$
Vì $p=2003\neq 8k-1\neq 8k+1$ suy ra $2$ không là SCP mod p $\Rightarrow 2^{\frac{p-1}{2}}\equiv -1(Modp))$
Mà $10^{7}\equiv 2^{10}(Modp)\Rightarrow 2.10^{700}\equiv 2^{1001}= 2^{\frac{p-1}{2}}\equiv -1 (Modp)$
$\Rightarrow (2.10^{700}+1)\vdots p$ .Mà $S(2.10^{700}+1)= 3$ nên khẳng định của ta là đúng.
Vậy $minS(n)= 3$.Xảy ra khi chẳng hạn $n= 2.10^{700}+1$



#379636 Cho $x,y,z>0$. CMR: $\sum\dfrac{2x^2+xy...

Đã gửi bởi Secrets In Inequalities VP on 22-12-2012 - 20:27 trong Bất đẳng thức và cực trị

Chuẩn hóa $x + y + z = 1$
Ta có $$\left (y + \sqrt{xz} + z\right )^2 \le \left (y + x + z\right )\left (y + z + z\right ) = (x + y + z)(y + 2z) = y + 2z$$
Nên $$VT \ge \dfrac{2x^2 + xy}{y + 2z} + \dfrac{2y^2 + yz}{z + 2x} + \dfrac{2z^2 + zx}{x + 2y} $$
Lại có $$\dfrac{2x^2 + xy}{y + 2z} + x(2x + y).(y + 2z) \ge 2\left (2x^2 + xy\right ) \Leftrightarrow \dfrac{2x^2 + xy}{y + 2z} \ge 2\left (2x^2 + xy\right ) - x(2x + y)(y + 2z) \ge 2\left (2x^2 + xy\right )$$ $$ - \dfrac{x(2x + 2y + 2z)^2}{4} = 2\left (2x^2 + xy\right ) - x$$
Từ đó, ta có :
$$VT \ge 2\left (\left (2x^2 + xy\right ) + \left (2y^2 + yz\right ) + \left (2z^2 + zx\right )\right ) - \left (x + y + z\right ) = 3\left (x^2 + y^2 + z^2\right )$$ $$ + \left (x + y + z\right )^2 - 1 \ge (x + y + z)^2 + 1 - 1 = 1$$
Suy ra ĐPCM.



#379628 $\sum \frac{1}{(a+2b)^2}\geq \fr...

Đã gửi bởi Secrets In Inequalities VP on 22-12-2012 - 20:11 trong Bất đẳng thức và cực trị

Bài 2/ Chứng minh rằng với mọi $a,b,c>0$
$\frac{1}{b^2+bc+c^2}+\frac{1}{c^2+ca+a^2}+\frac{1}{a^2+ab+b^2}\geq \frac{9}{(a+b+c)^2}$
(Cũng đã lâu rồi nhưng cũng rất hay, @Đạt: không cần chuẩn hóa và rất cổ điển)

Ta có : $\frac{1}{b^2+bc+c^2}= \frac{ab+bc+ca}{(b^2+bc+c^2)(ab+bc+ca)}\geq \frac{4(ab+bc+ca)}{(b^2+bc+c^2+ab+bc+ca)^2}$
$$= \frac{4(ab+bc+ca)}{(b+c)^2(a+b+c)^2}$$
Tương tự rồi cộng vào ta cần chứng minh :
$\frac{4(ab+bc+ca)}{(b+c)^2(a+b+c)^2}+\frac{4(ab+bc+ca)}{(c+a)^2(a+b+c)^2}+\frac{4(ab+bc+ca)}{(a+b)^2(a+b+c)^2}\geq \frac{9}{(a+b+c)^2}$
$\Leftrightarrow \frac{1}{(b+c)^2}+\frac{1}{(c+a)^2}+\frac{1}{(a+b)^2}\geq \frac{9}{4(ab+bc+ca)}$
Đúng theo $IRan96$



#379624 $\frac{1}{(a+1)^2}+\frac{1}...

Đã gửi bởi Secrets In Inequalities VP on 22-12-2012 - 19:59 trong Bất đẳng thức - Cực trị

Đề thi học kỳ bài khó thứ nhì trường THPT chuyên Hà Nội-Amsterdam Khá quen thuộc.
Bài toán: Cho$a,b,c$ là các số thực dương $abc=1.$. Chứng minh rằng:

$\frac{1}{(a+1)^2}+\frac{1}{(b+1)^2}+\frac{1}{(c+1)^2}+\frac{2}{(a+1)(b+1)(c+1)}\geq 1$

Ta thấy trong 3 số $a,b,c$ sẽ có 2 số cùng lớn hơn hoặc nhỏ hơn 1.Giả sử là $a$ và $b$.
$(a-1)(b-1)\geq 0\Leftrightarrow ab+1\geq a+b\Leftrightarrow 2(ab+1)\geq (a+1)(b+1)$
Áp dụng nhận xét trên kết hợp vs bổ đề qen thuộc $\frac{1}{(x+1)^2}+\frac{1}{(y+1)^2}\geq \frac{1}{xy+1}$ :
$VT\geq \frac{1}{ab+1}+\frac{1}{(c+1)^2}+\frac{2}{2(ab+1)(c+1)}= \frac{1}{\frac{1}{c}+1}+\frac{1}{(c+1)^2}+\frac{1}{(\frac{1}{c}+1)(c+1)}$
$$= \frac{c}{c+1}+\frac{1}{(c+1)^2}+ \frac{c}{(c+1)^2}$$
$$= \frac{c(c+1)}{(c+1)^2}+\frac{1}{(c+1)^2}+ \frac{c}{(c+1)^2}= \frac{(c+1)^2}{(c+1)^2}= 1$$
-------------
Hỵ hỵ hôm qua cũng nghĩ ra thêm cách này mà lười p0st :3 :luoi:



#378398 CMR: $\triangle ABC$ đều

Đã gửi bởi Secrets In Inequalities VP on 17-12-2012 - 21:16 trong Hình học

Bài toán: Chứng minh rằng, tam giác $ABC$ đều khi và chỉ khi
$$\frac{a\cdot\angle A + b\cdot\angle B + c\cdot\angle C}{a+b+c} = 60^\circ$$

Giả sử $a\geq b\geq c\Rightarrow \angle A\geq \angle B\geq \angle C$
Theo $chebyshev$ : $\frac{a\cdot\angle A + b\cdot\angle B + c\cdot\angle C}{a+b+c} \geq \frac{(a+b+c)(\angle A+\angle B+\angle B)}{3(a+b+c)}= 60$
Xét dấu bằng một chút là ta có $Q.E.D$



#375996 Chứng minh rằng tồn tại vô số n để $a_n$ chia hết cho 7

Đã gửi bởi Secrets In Inequalities VP on 08-12-2012 - 14:28 trong Dãy số - Giới hạn

Cho $\large{(a_n)\begin{cases}a_0=1\\a_n=a_{n-1}+a_{[\frac{n}{2}]}\end{cases}}$
Chứng minh rằng tồn tại vô số n để $a_n$ chia hết cho 7

Giả sử chỉ có hữu hạn số chia hết cho $7$ và $a_k$ là số cuối cùng trong các số ấy
Theo ct xác định dãy ta có : $a_{2k+1}= a_{2k}+a_k;a_{2k}= a_{2k-1}+a_k$
$\Rightarrow a_{2k-1}\equiv a_{2k}\equiv a_{2k+1}\equiv b \pmod 7$
Ta có : $a_{4k-3}\equiv a_{4k-3}+0.b \pmod 7$
$a_{4k-2}= a_{4k-3}+a_{2k-1}\equiv a_{4k-3}+1.b \pmod 7$
$a_{4k-1}= a_{4k-2}+a_{2k-1}\equiv a_{4k-3}+2.b \pmod 7$
$a_{4k}= a_{4k-1}+a_{2k}\equiv a_{4k-3}+3.b \pmod 7$
$a_{4k+1}= a_{4k}+a_{2k}\equiv a_{4k-3}+4.b \pmod 7$
$a_{4k+2}= a_{4k+1}+a_{2k+1} \equiv a_{4k-3}+5.b \pmod 7$
$a_{4k+3}= a_{4k+2}+a_{2k+1} \equiv a_{4k-3}+6.b \pmod 7$
Do $\gcd (b,7)=1$ nên các số $a_{4k-3}+ib$ với $i$ chạy từ $0$ đến $6$ lập thành 1 HĐĐ modulo 7.
Nên trong $7$ số $a_{4k-3}+ib$ phải có một số chia hết cho $7$, mà số này lại lớn hơn $a_k$: mâu thuẫn vs điều giả sử
Vậy giả sử là sai và ta có $Q.E.D$

p/s :Ai sửa hộ mình cái Latex vs .Thanks trước !

Perfectstrong: Đề nghị em nên học một lớp bổ túc về latex gấp :P



#375640 Chứng minh rằng phương trình $x^{4}+3y^{4}=z^{4...

Đã gửi bởi Secrets In Inequalities VP on 06-12-2012 - 20:38 trong Số học

Chứng minh rằng phương trình $x^{4}+3y^{4}=z^{4}$ có vô số nghiệm nguyên

Ta thấy : $k^{4}+3.0^{4}= k^4$
Vậy pt có vô số nghiệm nguyên $(x,y,z)=(k,0,k)$ với $k\in\mathbb{Z}$



#375309 Xác định giá trị $p,q$ để $S$ là một số nguyên tố

Đã gửi bởi Secrets In Inequalities VP on 05-12-2012 - 15:19 trong Số học

Ta có nhận xét là nếu $a \not\in\mathbb{Z}$ thì $[a]+[-a]= -1$
Xét : $[\frac{kq}{p}]+[\frac{(p-k)q}{q}]= [\frac{kq}{p}]+[q-\frac{kp}{q}]= q+[\frac{kq}{p}]+[-\frac{kq}{p}]$ $= q-1$
( vì do $(p,q)=1$ nên $\frac{kp}{q}\not\in\mathbb{Z}$ nên áp dụng NX trên thì ta có đc điều này )
Tương tự suy ra : $S= \frac{(p-1)(q-1)}{2}$
Dễ rồi !



#375185 Tìm tất cả các cấp số cộng có vô hạn số hạng sao cho tồn tại số $N$...

Đã gửi bởi Secrets In Inequalities VP on 04-12-2012 - 21:25 trong Số học

Tìm tất cả các cấp số cộng có vô hạn số hạng sao cho tồn tại số $N$ mà với mọi $p>N$, nếu $a_p$ nguyên tố thì $p$ cũng là số nguyên tố



#374340 $\frac{a}{a^2+ab+bc}+\frac{b}...

Đã gửi bởi Secrets In Inequalities VP on 01-12-2012 - 21:05 trong Bất đẳng thức và cực trị

Bài 6: Cho các số thực $a,b,c>0$ sao cho $abc=1$, chứng minh rằng: $$(a+b)(b+c)(c+a)\geq 4(a+b+c-1)$$

Thêm 1 bài cuối nhé, tờ giấy bị rách câu 6 mới tìm lại được :icon6:

Lâu lâu chém tí BĐT cho vui ! ^^
Áp dụng bổ đề quen thuộc :$(a+b)(b+c)(c+a)\geq \frac{8}{9}(ab+bc+ca)(a+b+c)$
Ta chỉ cần phải CM : $\frac{8}{9}(ab+bc+ca)(a+b+c)\geq 4(a+b+c-1)$
Chia cả 2 vế cho $a+b+c$ ta đc BĐT tương đương là :
$$\frac{8}{9}(ab+bc+ca)+\frac{4}{a+b+c}\geq 4$$
Đúng theo $AM-GM$ :
$VT= \frac{4}{9}(ab+bc+ca)+\frac{4}{9}(ab+bc+ca)+\frac{4}{a+b+c}\geq 3\sqrt[3]{\frac{4^{3}}{9^{2}}\frac{(ab+bc+ca)^2}{a+b+c}}$
$$3\sqrt[3]{\frac{4^{3}}{9^{2}}\frac{(ab+bc+ca)^2}{a+b+c}}\geq 3\sqrt[3]{\frac{4^{3}}{9^{2}}.\frac{3abc(a+b+c)}{a+b+c}}= 4$$
Vậy ta có $Q.E.D$



#374311 Chứng minh rằng có vô số số nguyên tố dạng: $p=8k+1$

Đã gửi bởi Secrets In Inequalities VP on 01-12-2012 - 20:27 trong Số học

Chứng minh rằng có vô số số nguyên tố dạng:
a) $p=8k+1$

Tổng quát : Với mỗi số n nguyên dương cho trước luôn tồn tại vô số số nguyên tố dạng $2^{n}k+1$
Ta dùng bổ đề :
Bổ đề : Cho $a,n\in\mathbb{N}^{*}$ .Khi đó mọi ước nguyên tố p của số $a^{2^{n}}+1$ hoặc bằng $2$ hoặc có dạng $p= 2^{n+1}k+1$
Vào bài toán :
Giả sử chỉ có hữu hạn số ntố dạng $2^{n}k+1$ là ${p_1},{p_2},..,{p_m}$
Đặt $a=2{p_1}{p_2}...{p_m}$
Xét số $A= a^{2^{n-1}}+1$ .Theo bổ đề thì mọi ước ntố p của $A$ đều có dạng $p= 2^{n}k+1$.
Số $p$ này phải khác tất cả các ${p_i}$ vì nếu $p= {p_i}$ nào đó thì $a\vdots p$ mà $A\vdots p$ $\Rightarrow 1\vdots p$ vô lí
$\Rightarrow p\neq {p_i}$
Mà $p= 2^{n}k+1$. nên mâu thuẫn vs giả sử $\Rightarrow$ giả sử sai $\Rightarrow Q.E.D$



#372827 $x^{2009}+y^{2009}=7^{z}$

Đã gửi bởi Secrets In Inequalities VP on 26-11-2012 - 19:37 trong Số học

Tìm x,y,z tự nhiên thỏa mãn:
$x^{2009}+y^{2009}=7^{z}$

Vì $2009= 7^{2}.41$ và $x+y|x^{2009}+y^{2009}$ nên dễ thấy $7|x+y$
Theo LTE : ${v_p}(x^{2009}+y^{2009})= {v_p}(x+y)+{v_p}(2009)= {v_p}(x+y)+2$
$\Rightarrow x^{2009}+y^{2009}= 49k(x+y)$
Mà $x^{2009}+y^{2009}=7^{z}$ là luỹ thua của 7 nên suy ra $k=1$
$\Rightarrow x^{2009}+y^{2009}= 49(x+y)$
Dễ thấy rằng nếu $(x,y)\neq (1,1)$ thì $VT> VP$
Lại dễ thấy $(1,1)$ không thỏa mãn đễ nên pt vô nghiệm !



#372364 $a^{2^{n}}+b^{2^{n}}$

Đã gửi bởi Secrets In Inequalities VP on 25-11-2012 - 10:33 trong Số học

Giải trí nhân ngày 26/11 (ngày mai ) :P ! Cũng dễ thôi !
Cho $a,b \in\mathbb{N}^{*}$ , $(a,b)=1$. CMR : Mọi uoc nguyên tố lẻ của$a^{2^{n}}+b^{2^{n}}$ đều có dạng $2^{n+1}k+1$



#372357 $a^{3}b^{3}+b^{3}c^{3}+c^{3...

Đã gửi bởi Secrets In Inequalities VP on 25-11-2012 - 10:21 trong Bất đẳng thức - Cực trị

Cho các số không âm a,b,c thỏa a+b+c=2. Chứng minh rằng:
$a^{3}b^{3}+b^{3}c^{3}+c^{3}a^{3}\leq 1$

Lâu lâu ms làm bđt ! :biggrin:
Giả sủ $a=max(a,b,c)$
Dễ thấy $3a^3(b+c)\geq b^2c^2\Rightarrow 3a^3bc(b+c)\geq b^3c^3$
$\Rightarrow VT\leq a^3b^3+3a^3bc(b+c)+a^3c^3= a^3(b+c)^3$
$a(b+c)\leq \frac{[[a+(b+c)]]^2}{4}= 1\Rightarrow VT\leq 1\Rightarrow Q.E.D$



#372155 $\widehat {BOC} = \widehat {AOD}$

Đã gửi bởi Secrets In Inequalities VP on 24-11-2012 - 19:59 trong Hình học

Cho tứ giác lồi ABCD.Gọi $E = AB \cap CD;F = AD \cap BC;P = AC \cap BD$.GỌI O là hình chiếu của P trên EF.CMR$\widehat {BOC} = \widehat {AOD}$

+ TH1: $BD//EF$ dễ thấy $Q.E.D$

+ TH2: $BD$ không song song với $EF$

$S= BC\cap EF;K=BP\cap AF$

$\Rightarrow (ADKF)= -1\Rightarrow (ACPS)= -1$ ( chiếu xuyên tâm E )

$\Rightarrow O(ACPS)= -1$.

$OP \bot OS \Rightarrow \widehat{POA} = \widehat{POC}$

Ta có : $(ADKF)= -1 \Rightarrow E(ADKF)= -1 \Rightarrow E(BDPF)= -1 \Rightarrow O(BDPF)= -1$.

$OP \bot OF \Rightarrow \widehat{POB} = \widehat{POF}$

Kết hop 2 cái màu đỏ suy ra :

$\widehat{BOA}= \widehat{DOC} \Rightarrow \widehat{BOC}=\widehat{DOA}= \widehat{AOD} \Rightarrow Q.E.D$

p/s: thanks a Hân ! :lol:



#371838 CMR: $AA_{1}=BA_{1}$

Đã gửi bởi Secrets In Inequalities VP on 23-11-2012 - 19:01 trong Hình học

Bài toán
Cho các điểm$A_{1},B_{_{1}}$ theo thứ tự thuộc CB,CA của tam giác ABC. D=$A_{1}A\cap BB_{_{1}}=D;A_{1}B_{1}\cap CD=E.$ Biết rằng $\measuredangle A_{1}EC=90$ và $A,B,E,A_{1}$ cùng thuộc 1 đường tròn. CMR: $AA_{1}=BA_{1}$


$AE\cap BC= F$.$\Rightarrow (BFAC)= -1\Rightarrow E(BFAC)= -1$
Mà $EA\perp EC\Rightarrow \angle AEB= \angle AEF\Rightarrow \angle {A_1}BA= \angle AEF= \angle AEB={A_1}AB $
$\Rightarrow \Delta A{A_1}B $ cân tại ${A_1}$ $\Rightarrow A{A_1}= B{A_1}$



#371033 Tính tổng $\sum_{2012}^{4024}p\left ( n...

Đã gửi bởi Secrets In Inequalities VP on 20-11-2012 - 20:21 trong Số học

Với mỗi số tự nhiên $n$, gọi $p(n)$ là ước số lẻ lớn nhất của $n$. Hãy tính tổng $\sum_{n=2012}^{4024}p\left ( n \right )$.

Đặt $v(n)$ là số mũ cao nhất của $2$ trong $n$ . Chú ý $v(n)p(n)= n\Rightarrow p(n)= \frac{n}{v(n)}$ là ra !



#369761 $\sum _{0\leq k\leq n;(k,n)=1}k= \frac...

Đã gửi bởi Secrets In Inequalities VP on 15-11-2012 - 22:27 trong Số học

Cho $n$ là số tự nhiên lớn hơn 1 . Chứng minh rằng : $\sum _{0\leq k\leq n;(k,n)=1}k= \frac{1}{2}n\phi (n)$

Bài này chỉ cần một nhận xét nhỏ là solved problem :P :
Dễ thấy nếu $gcd(k,n)= 1$ thì $gcd(n,n-k)= 1$
$\Rightarrow \sum _{0\leq k\leq n;(k,n)=1}k= \sum _{0\leq k\leq n;(k,n)=1}(n-k)$$\Rightarrow \sum _{0\leq k\leq n;(k,n)=1}k= \frac{1}{2}( \sum _{0\leq k\leq n;(k,n)=1}k+ \sum _{0\leq k\leq n;(k,n)=1}(n-k))= \frac{1}{2}\phi (n)n$
( vì số các số k mà $gcd(k,n)=1$ bằng $\phi (n)$ )



#369746 $S = C_{2n + 1}^0{.2^{2n}} + C_{2n +...

Đã gửi bởi Secrets In Inequalities VP on 15-11-2012 - 21:56 trong Số học

Đặt $a=1+\sqrt{3},b= 1-\sqrt{3}\Rightarrow ab=-2;\frac{a^2}{2}= 2+\sqrt{3};\frac{b^2}{2}= 2-\sqrt{3}$
${P_n}= \frac{1}{2}(a^{2n+1}+b^{2n+1})$
Dùng khai triển Newton cho $(1+\sqrt{3})^{2n+1},(1-\sqrt{3})^{2n+1}$ta suy ra đc ${P_n}= \sum_{k=0}^{n}\binom{2k}{2n+1}.3^k$ là số nguyên .
Lại dùng Newton cho $(2+\sqrt{3})^{2n+1},(2-\sqrt{3})^{2n+1}$ ta đc:
${S_n}= \frac{(\frac{a^2}{2})^{2n+1}+(\frac{b^2}{2})^{2n+1}}{4}= \frac{a^{4n+2}+b^{4n+2}}{2^{2n+3}}= \frac{a^{4n+2}+2(ab)^{2n+1}+b^{4n+2}}{2^{2n+3}}+\frac{1}{2}= \frac{{P_n}^{2}}{2^{2n+1}}+\frac{1}{2}$
Nhân chéo lên
$\Rightarrow {P_n}^{2}\vdots 2^{2n},{P_n}^{2}\not\vdots 2^{2n+1}\Rightarrow {P_n}^{2}= m2^{2n}$ ( m lẻ )
Mà ${P_n}^{2}$ và $2^{2n}$ chính phuong nên $m= (2p+1)^{2}$
$\Rightarrow {P_n}^{2}= (2p+1)^22^{2n}$
$\Rightarrow {P_n}= (2p+1)2^{^{n}}\Rightarrow\frac{{P_n}-2^n}{2^{n+1}}= p$
Mà dễ thấy :
${S_n}= \frac{{P_n}^{2}}{2^{2n+1}}+\frac{1}{2}= (\frac{{P_n}-2^n}{2^{n+1}})^{2}+(\frac{{P_n}+2^n}{2^{n+1}})^{2}= p^{2}+(p+1)^{2}$
Vậy ta có $Q.E.D$